Any of the following could be true EXCEPT:

Sidrah on August 21, 2019

Please explain

Could you please explain why B is the correct answer? Also, would it be possible to get video explanations for the logic game section of the prep tests?

Reply
Create a free account to read and take part in forum discussions.

Already have an account? log in

Irina on August 21, 2019

@Sidra,

Please direct any content-related feedback to our support team. You may reach them by tapping "support" from the left menu or by calling 855.483.7862 ext. 2 Monday-Friday 9am-6pm PT.

Let's look at the setup for this game.

A travel agent is arranging a tour visiting exactly 4 of the 6 cities - H, J, M, O, S, T.
Each city will be visited only once.
___ ___ ___ ___
1 2 3 4

The tour's schedule is subject to the following constraints:

(1) H & T must be included in the tour but they cannot be visited consecutively.
~ HT
~TH

(2) If O is included, S cannot be.

O -> ~S
S -> ~O

This rule tells us that only one of the cities could be included either O or S, not both. Both could be excluded though.

(3) If J is included, it must be #3

J -> 3

(4) If both J & M are included, they must be visited consequently

J & M -> JM v MJ

Combined with rule (3) and rule (1), we can infer that if JM are included, J must be #3, and M must be #2, otherwise, T & H will be consecutive in violation of rule (1):

/H /H
/T /T
___ M J ___
1 2 3 4

We can also infer any scenario when J is included and is #3 per rule (3), H or T must be #4, otherwise, they would be consecutive in violation of rule (1).

Let's look at the answer choices:

(A) J is visited immediately after H is visited

O/S H J T
1 2 3 4

This is a valid order, thus (A) is incorrect.

(B) M is visited some time after J is visited.

We can tell right away by looking at our initial setup that if J is visited, #4 must be H or T, thus, it is impossible for M to be #4. Thus, this is the correct answer choice.

(C) O is included in the tour but visited third.

O H J T
1 2 3 4

This is one of the valid scenarios, thus (C) is incorrect.

(D) M is the only city visited between H & T.

O H M T
1 2 3 4

This is one of the valid scenarios, thus (D) is incorrect.

(E) More than one city is visited between H & T.

H M J T
1 2 3 4

This is one of the valid scenarios, thus (E) is incorrect.

Does this make sense?

Let me know if you have any further questions.